Difference between revisions of "2014 AMC 12B Problems/Problem 25"
Kevin38017 (talk | contribs) m (→Solution) |
Johnstucky (talk | contribs) m (→Solution) |
||
Line 12: | Line 12: | ||
Notice that either <math>a = 1</math> and <math>b = 1</math> or <math>a = -1</math> and <math>b = -1</math>. For the first case, <math>a = 1</math> only when <math>x = k\pi</math> and <math>k</math> is an integer. <math>b = 1</math> when <math>\frac{2014\pi^2}{k\pi}</math> is an even multiple of <math>\pi</math>, and since <math>2014 = 2*19*53</math>, <math>b =1</math> only when <math>k</math> is an odd divisor of <math>2014</math>. This gives us these possible values for <math>x</math>: | Notice that either <math>a = 1</math> and <math>b = 1</math> or <math>a = -1</math> and <math>b = -1</math>. For the first case, <math>a = 1</math> only when <math>x = k\pi</math> and <math>k</math> is an integer. <math>b = 1</math> when <math>\frac{2014\pi^2}{k\pi}</math> is an even multiple of <math>\pi</math>, and since <math>2014 = 2*19*53</math>, <math>b =1</math> only when <math>k</math> is an odd divisor of <math>2014</math>. This gives us these possible values for <math>x</math>: | ||
<cmath>x= \pi, 19\pi, 53\pi, 1007\pi</cmath> | <cmath>x= \pi, 19\pi, 53\pi, 1007\pi</cmath> | ||
− | For the case where <math>a = -1</math>, <math>\cos{2x} = -1</math>, so <math>x = \frac{m\pi}{2}</math>, where m is odd. <math>\frac{2014\pi^2}{\frac{m\pi}{2}}</math> must also be an odd multiple of \pi in order for <math>b</math> to equal <math>-1</math>, so <math>\frac{4028}{m}</math> must be odd. We can quickly see that dividing an even number by an odd number will never yield an odd number, so there are no possible values for <math>m</math>, and therefore no cases where <math>a = -1</math> and <math>b = -1</math>. Therefore, the sum of all our possible values for <math>x</math> is | + | For the case where <math>a = -1</math>, <math>\cos{2x} = -1</math>, so <math>x = \frac{m\pi}{2}</math>, where m is odd. <math>\frac{2014\pi^2}{\frac{m\pi}{2}}</math> must also be an odd multiple of <math>\pi</math> in order for <math>b</math> to equal <math>-1</math>, so <math>\frac{4028}{m}</math> must be odd. We can quickly see that dividing an even number by an odd number will never yield an odd number, so there are no possible values for <math>m</math>, and therefore no cases where <math>a = -1</math> and <math>b = -1</math>. Therefore, the sum of all our possible values for <math>x</math> is |
<cmath>\pi + 19\pi + 53\pi + 1007\pi = \boxed{\textbf{(D)}\ 1080 \pi}</cmath> | <cmath>\pi + 19\pi + 53\pi + 1007\pi = \boxed{\textbf{(D)}\ 1080 \pi}</cmath> | ||
(Solution by kevin38017) | (Solution by kevin38017) |
Revision as of 19:08, 20 February 2014
Problem
Find the sum of all the positive solutions of
$\textbf{(A)}\ \pi \qquad\textbf{(B)}\ 35\qquad\textbf{(C)}\ 1008\pi \qquad\textbf{(D)}}\ 1080 \pi \qquad\textbf{(E)}\ 1800\pi$ (Error compiling LaTeX. Unknown error_msg)
Solution
Rewrite as . Now let , and let . We have Notice that either and or and . For the first case, only when and is an integer. when is an even multiple of , and since , only when is an odd divisor of . This gives us these possible values for : For the case where , , so , where m is odd. must also be an odd multiple of in order for to equal , so must be odd. We can quickly see that dividing an even number by an odd number will never yield an odd number, so there are no possible values for , and therefore no cases where and . Therefore, the sum of all our possible values for is
(Solution by kevin38017)